1992 AJHSME Problems/Problem 2

Revision as of 21:31, 3 October 2009 by 5849206328x (talk | contribs) (Created page with '==Problem== Which of the following is not equal to <math>\dfrac{5}{4}</math>? <math>\text{(A)}\ \dfrac{10}{8} \qquad \text{(B)}\ 1\dfrac{1}{4} \qquad \text{(C)}\ 1\dfrac{3}{12}…')
(diff) ← Older revision | Latest revision (diff) | Newer revision → (diff)

Problem

Which of the following is not equal to $\dfrac{5}{4}$?

$\text{(A)}\ \dfrac{10}{8} \qquad \text{(B)}\ 1\dfrac{1}{4} \qquad \text{(C)}\ 1\dfrac{3}{12} \qquad \text{(D)}\ 1\dfrac{1}{5} \qquad \text{(E)}\ 1\dfrac{10}{40}$

Solution

The fraction in question is equal to $1.25$. Expressing all of the choices we have,

$\text{(A)}\ 1.25 \qquad \text{(B)}\ 1.25 \qquad \text{(C)}\ 1.25 \qquad \text{(D)}\ 1.2 \qquad \text{(E)}\ 1.25$.

It's clear then that choice $\boxed{\text{D}}$ is the answer.

See Also

1992 AJHSME (ProblemsAnswer KeyResources)
Preceded by
Problem 1
Followed by
Problem 3
1 2 3 4 5 6 7 8 9 10 11 12 13 14 15 16 17 18 19 20 21 22 23 24 25
All AJHSME/AMC 8 Problems and Solutions